Membatasi distribusi

8

Biarkan menjadi urutan variabel acak iid . Tetapkan dan untuk . Temukan distribusi pembatas dari(Xn)N(0,1)S0=0Sn=k=1nXkn1

1nk=1n|Sk1|(Xk21)

Masalah ini berasal dari sebuah buku masalah tentang Teori Probabilitas, dalam bab tentang Central Limit Theorem.

Karena dan independen, danSk1XkE(|Sk1|(Xk21))=0

V(|Sk1|(Xk21))=E(Sk12(Xk21)2)=E(Sk12)E(Xk21)2)=2(k1)

Perhatikan bahwa jelas tidak independen. Masalahnya adalah dari Masalah Shiryaev dalam Probabilitas , yang dengan sendirinya didasarkan pada buku teks dari penulis yang sama. Buku teks tampaknya tidak mencakup CLT untuk variabel berkorelasi. Saya tidak tahu apakah ada stasioner, campuran urutan bersembunyi di suatu tempat ...|Sk1|(Xk21)

Saya telah menjalankan simulasi untuk merasakan jawabannya

import numpy as np
import scipy as sc
import scipy.stats as stats
import matplotlib.pyplot as plt

n = 20000 #summation index
m = 2000 #number of samples

X = np.random.normal(size=(m,n))
sums = np.cumsum(X, axis=1)
sums = np.delete(sums, -1, 1)
prods = np.delete(X**2-1, 0, 1)*np.abs(sums)
samples = 1/n*np.sum(prods, axis=1)

plt.hist(samples, bins=100, density=True)
x = np.linspace(-6, 6, 100)
plt.plot(x, stats.norm.pdf(x, 0, 1/np.sqrt(2*np.pi)))
plt.show()

Di bawah ini adalah histogram dari sampel ( ). Itu terlihat terbagi normal ...2000n=20.000

masukkan deskripsi gambar di sini

Gabriel Romon
sumber
@ MartijnWeterings Saya memposting ini karena saya telah merenungkan masalah selama beberapa waktu dan saya mandek. Itu mungkin jauh dari sepele ...
Gabriel Romon
@MartijnWeterings , makaE(|Sk1|(Xk21))=0V(|Sk1|(Xk21))=E(Sk12(Xk21)2)
Gabriel Romon
@ MartijnWeterings Ya, saya menghilangkan persamaan sepele untuk ...|x|2=x2xR
Gabriel Romon
Histogram dalam simulasi sangat cocok untuk distribusi Normal. Jika Anda tidak yakin, hitung kurtosis.
Whuber
@ MartijnWeterings Ya, saya membuat penghilangan kode yang memalukan. Saya telah memperbaruinya, serta histogram, yang terlihat seperti yang normal. Apakah Anda memiliki gagasan tentang nilai varians yang tepat?
Gabriel Romon

Jawaban:

1

Ketika saya mensimulasikan distribusi maka saya mendapatkan sesuatu yang menyerupai distribusi Laplace. Bahkan lebih baik tampaknya menjadi q-Gausian (parameter tepat yang harus Anda temukan menggunakan teori).

Saya kira bahwa buku Anda harus mengandung beberapa variasi CLT yang berhubungan dengan itu (teorema limit pusat yang digeneralisasi, mungkin dalam Bagian 7.6 Teorema batas pusat untuk jumlah variabel dependen , tetapi saya tidak dapat mencarinya karena saya tidak memiliki buku yang tersedia).

simulasi

library(qGaussian)
set.seed(1)
Qstore <- c(0) # vector to store result

n <- 10^6  # columns X_i
m <- 10^2  # rows repetitions

pb <- txtProgressBar(title = "progress bar", min = 0,
                     max = 100, style=3)
for (i in 1:100) {  
  # doing this several times because this matrix method takes a lot of memory
  # with smaller numbers n*m it can be done at once

  X <- matrix(rnorm(n*m,0,1),m)
  S <- t(sapply(1:m, FUN = function(x) cumsum(X[x,])))
  S <- cbind(rep(0,m),S[,-n])
  R <- abs(S)*(X^2-1)
  Q <- t(sapply(1:m, FUN = function(x) cumsum(R[x,])))

  Qstore <- c(Qstore,t(Q[,n]))
  setTxtProgressBar(pb, i)
}
close(pb)

# compute histogram 
x <- seq(floor(min(Qstore/n)), ceiling(max(Qstore/n)), 0.2)
h <- hist(Qstore/(n),breaks = x)

# plot simulation
plot( h$mid, h$density, log = "y", xlim=c(-7,7),
      ylab = "log density" , xlab = expression(over(1,n)*sum(abs(S[k-1])*(X[k]^2-1),k==1,n) ) )

# distributions for comparison
lines(x, dnorm(x,0,1),                   col=1, lty=3)      #normal 
lines(x, dexp(abs(x),sqrt(2))/2,         col=1, lty=2)      #laplace
lines(x, qGaussian::dqgauss(x,sqrt(2),0,1/sqrt(2)), col=1, lty=1)      #qgauss

# further plotting
title("10^4 repetitions with n=10^6")
legend(-7,0.6,c("Gaussian", "Laplace", "Q-Gaussian"),col=1, lty=c(3,2,1),cex=0.8)
Sextus Empiricus
sumber
Mengenai konten buku teks, yang terbaik adalah yang Anda lihat sendiri: Volume 1 , Volume 2 . Masalahnya seharusnya hanya memerlukan materi yang dibahas dalam Bab 3.4
Gabriel Romon
@GabrielRomon terima kasih banyak atas tautannya. Melihatnya, dari ponsel saya, saya tidak dapat menemukan apa pun tentang q-Gaussian atau distribusi terbatas lainnya yang bukan distribusi normal. Jadi distribusinya memiliki konvergensi yang sangat lambat n >> 10 ^ 6 sebelum kita melihatnya , atau pertanyaannya tidak sesuai dengan bab (apakah itu dari buku, saya tidak bisa menemukan pertanyaan?). Plot momen orde yang lebih tinggi (sebagai fungsi n) mungkin menunjukkan lebih baik apakah konversi mungkin masih terjadi, tapi saya rasa ini bukan kasus CLT biasa.
Sextus Empiricus
1
Ini adalah Soal 3.4.14 dalam buku masalah .
Gabriel Romon